[obm-l] Mais um membro pra lista

2002-10-01 Por tôpico Domingos Jr.
Olá, meu nome é Domingos, faço Ciências da Computação no IME.USP e pretendo ser mais um membro dessa prestigiada lista! Eu gostaria de ver se alguém pode me ajudar com uma questão... Essa é da olimpíada do Cone Sul: Dizemos que um inteiro n, n 1, é

Re: [obm-l] Mais um membro pra lista

2002-10-01 Por tôpico Domingos Jr.
se existe, só pode ser um: p1 + p2 + ... + pn n.pn (pois pn p[n-1] ... p1) se pk pn é tal que pk | p1 + ... + pn, existe q inteiro tal que pk.q = p1 + ... + pn q = (p1 + ... + pn)/pk n.(pn/pk) n pn e n pn, logo não há nenhum outro primo maior que pn que divide n... eu vou pensar

Re: [obm-l] Mais um membro pra lista

2002-10-02 Por tôpico Domingos Jr.
Ok... Agora ficou mais claro. De qualquer forma, o enunciado original pede um número ensolarado com um N específico (10^2000, se não me engano). sim, e isso já está resolvido, pega os 10^4000 primeiros primos, ou eles são tipo (1) ou (2), de qualquer forma podemos obter números ensolarados com

Fw: [obm-l] Teorema de Donald ; Olimpiada Iberoamericana de Matematica 2002

2002-10-09 Por tôpico Domingos Jr.
Agora que eu fui reparar, mas não são fatoriais... são quadrados!!! Se temos N homens e N mulheres existem N² possíveis pares, não N!. De qualquer modo, acho q o raciocínio vale... Ignorem a mensagem anterior... +-+ se N = 1 temos um

Re: [obm-l] Probabilidade

2002-10-14 Por tôpico Domingos Jr.
sejam a, b e c os tamanhos a + b c logo temos 3.P[a + b c] a probabilidade a ser calculada (o fator 3 vem do fato de que eu posso ter a + b c ou a + c b ou b + c a, tanto faz) esse tipo de probabilidade é resolvido em geral através deprobabilidadecondicional... no caso discreto seria

Re: [obm-l] Problemas

2002-10-15 Por tôpico Domingos Jr.
1) seja a = 10 + a0 e b = 10 + b0 com 0 = a0, b0 = 9 a.b = (10 + a0).(10 + b0) = 10.(10 + a0) + 10b0 + a0b0 = 10a + 10b0 + a0b0 = 10(a + b0) + a0b0 2) maior inteiro de n algarismos 99...9 emenor inteiro de n algarismos 1...0 - Original Message - From:

Re: [obm-l] gaucha

2002-10-19 Por tôpico Domingos Jr.
1) suponha que Carla aposta x. ou x está entre 1 e 32, ou entre 34 e 74 ou entre 76 e 100. caso 1 ~ 32 é conveniente colocar no número 31 ou 32, pois aí todo número n = 32 sorteado faz com que Carla ganhe. probabilidade 0,32 de Carla ganhar. caso 76 ~ 100 é conveniente apostar no número

Re: [obm-l] ??

2002-10-19 Por tôpico Domingos Jr.
1/x + 1/y = (x + y)/xy = 1/1998 suponha x, y 0 se d = mdc(x, y) [(x + y)/d]/[xy/d] é a forma irredutível do racional e, como 1/1998 já está na forma irredutível: (x + y)/d = 1 = d = x + y uma contradição, pois se d|x, (x + y)|x e x + y x... logo um deles é negativo... para x y 0 1/y -

RE: [obm-l] ??

2002-10-21 Por tôpico Domingos Jr.
Não entendi! Vc está querendo dizer que não existe x e y positivos para 1/x + 1/y = 1/1998 ?? E a solução trivial x=2*1998 e y=2*1998? --- x --- eu me equivoquei nesta parte: se d = mdc(x, y) [(x + y)/d]/[xy/d] é a forma irredutível do racional isso está errado! eu tinha mandado uma

Re: [obm-l] OBM-u

2002-10-22 Por tôpico Domingos Jr.
Questão 2. Uma matriz quadrada n por n tem diagonal por formada por 1s e as somas dos módulos dos elementos de cada linha não é maior do que 2. Mostre que o determinante está entre 0 (inclusive) e 1, não podendo ser 1. Se A for tal matriz podemos calcular sua norma 2 de uma maneira não mto

Re: [obm-l] OBM-u

2002-10-22 Por tôpico Domingos Jr.
os elementos da matriz são todos menores ou iguais a 1 em módulo, com a igualdade valendo para toda a diagonal, isso nos dá a noção intuitiva que ||A|| = 1 parece que a minha intuição não tá mto boa hoje! no entanto acho que o que foi proposto está errado! tome A = | 10.990 | | 0

Re: [obm-l] E o Nivel Tres,ninguem faz nada??????

2002-10-25 Por tôpico Domingos Jr.
ah, essa é legal... pegue p e q primos absurdamente gigantes! S = {2.p^q, 3p^q, , 2003.p^q} é um conjunto com 2002 inteiros positivos sendo que qualquer soma entre eles dá um número k.p^q onde 2 =k = 2+3+...+2003 como q é primo, a única maneira de k.p^q ser uma potência perfeita é se

Re: [obm-l] Teorema de Donald

2002-10-17 Por tôpico Domingos Jr.
Um algoritmo feito assim pode varrer todas as combinações de pares possíveis se isso for necessário (acho que é altamente improvável que isso ocorra, talvez seja atéimpossível). a demonstração que eu tinha imaginado para o problema (logo após a sua primeira postagem) era +/- assim: para n

Re: [obm-l] E o Nivel Tres,ninguem faz nada??????

2002-10-26 Por tôpico Domingos Jr.
Eu também tive uma idéia para o problema 3: No caso 2x2 é fácil ver que existe um elemento i vizinho de i+3, supondo (um argumento de indução) que isso vale para guaquer tabuleiro de dimensões até m x n, pegamos um tabuleiro maior que m x n. Neste tabuleiro podemos visualizar um caminho de

Re: [obm-l] Calorimetria ?

2002-10-28 Por tôpico Domingos Jr.
considere uma piscina de L litros com temperatura P. considere agora uma mangueira de vazão V, jorrando água de temperatura M na piscina. após a mangueira ter enchido a piscina com F litros, qual é a temperatura final da água !? seja T a temperatura final e Q o calor trocado (estou usando

Re: [obm-l] Olimp. do RN

2002-10-30 Por tôpico Domingos Jr.
300X - 198Y = 500 é uma eq. diofantina que não tem sol. pois mdc(300, 198) = 6 e 6 não divide 500 498 é o múltiplo de 6 mais perto de 500, e 300X - 198Y = 498 tem soluções x = 34, y = 49 sendo r, uma retirada e d um depósito, temos (r d r d d) repetido 16 vezes + (r d r) saldo final: $2, total

Re: [obm-l] Olimp. do RN

2002-10-31 Por tôpico Domingos Jr.
300Y - 198X = 500 é uma eq. diofantina sem solução pois mdc(300, 198) = 6 e 6 não divide 500, o múltiplo de 6 mais perto de 500 é 498 300X - 198Y = 498 50X - 33Y = 83, com X, Y 0 x = 34, y = 49 é solução dessa eq. fazendo 49 depósitos de $198 e 34 retiradas de $300 saldo final = 500 + 49*198 -

Re: [obm-l] Ajuda em probabilidade

2002-11-04 Por tôpico Domingos Jr.
hmmm, isso me lembra uns exercícios de processos estocásticos. podemos considerar os estados como simplesmente a distância entre o número de caras e de coroas, sendo que é fácil verificar a probabilidade em que a distância aumenta ou diminui. queremos verificar a probabilidade de num tempo

[obm-l] Re: [obm-l] TEORIA DOS NÚMEROS: PROBLEMA

2002-11-05 Por tôpico Domingos Jr.
(a-2)(a-1)a(a+1) = a(a²-1)(a-2) = (a³- a)(a-2) = a^4 - 2a³ - a² + 2a a^4 - 2a³ - a² + 2a + 1 = (a² - a - 1)² pra chegar nessa fatoração: (a² + d.a + e)² = a^4 + 2da³ + (2e + d²)a² + 2de.a + e² e² = 1 logo e = 1, -1 2de = 2, logo d = 1, -1 2e + d² = -1 logo e = -1 2d = -2 d = -1

[obm-l] Re: [obm-l] Re:_[obm-l]_TEORIA_DOS_NÚMEROS:_PROBLEMA

2002-11-05 Por tôpico Domingos Jr.
lugar aquele processo de fatoração ou criaste na resolução deste problema? "Domingos Jr." [EMAIL PROTECTED] wrote: (a-2)(a-1)a(a+1) = a(a²-1)(a-2) = (a³- a)(a-2) = a^4 - 2a³ - a² + 2aa^4 - 2a³ - a² + 2a + 1 = (a² - a - 1)²pra chegar nessa fatoração:(a² + d.a + e)² =

Re: [obm-l] P e NP

2002-11-10 Por tôpico Domingos Jr.
Basicamente problemas da classe P são aqueles para os quais existe um algoritmo que determina a(s) solução(ões) em tempo polinomial, problemas NP são aqueles problemas considerados difíceis pois não existe solução polinomial, só exponencial. Não-determinístico quer dizer que envolve algo

Re: [obm-l] subconjuntos

2002-11-11 Por tôpico Domingos Jr.
seja N = 2k + s, com s = {0, 1} 0) você pode formar um subconjunto vazio e outro com 2k+s elementos 1) um subconjunto com 1 elemento, outro com 2k+s-1 ... i) um com i elementos e outro com 2k+s-i se i k estamos contando alguns subconjuntos duas vezes, logo pegamos i = k seja S a soma de

Re: [obm-l] subconjuntos

2002-11-11 Por tôpico Domingos Jr.
é verdade, eu fui no embalo da outra resposta, a do Bruno Lima.. sendo assim, teremos uma somatória dupla que não é muito complicada de ser determinada. - Original Message - From: Augusto César Morgado To: [EMAIL PROTECTED] Sent: Monday, November 11, 2002 2:03 PM

Re: [obm-l] Teoremas

2002-11-12 Por tôpico Domingos Jr.
Existem demonstrações geométricas do teorema de Pitágoras, fica difícil postar na lista, mas eu mando um link para você: http://www.exatas.com/matematica/pitagoras.html#teorema Só um aviso, o teorema de Pitágoras vale para quaisquer valores, não só valores inteiros eo tma. de Fermat lida

[obm-l] Re: [obm-l] Re: [obm-l] equação

2002-11-14 Por tôpico Domingos Jr.
2) se a0 != 1 tome x = q.a0 com q inteiro, q != 0 fica simples verificar que a0|f(x) e, como f(x)= a0 = x é raiz de um polinômio de grau n (a1x +a2x^2 + anx^n) e esse polinômio pode ter no máximo n-1 raízes da forma q.a0 (uma raiz é 0), temos infinitas formas de escolher x = q.a0 de

[obm-l] Re: [obm-l] Re: [obm-l] equação

2002-11-14 Por tôpico Domingos Jr.
2) se a0 != 1 tome x = q.a0 com q inteiro, q != 0 fica simples verificar que a0|f(x) e, como f(x)= a0 = x é raiz de um polinômio de grau n (a1x +a2x^2 + anx^n) e esse polinômio pode ter no máximo n-1 raízes da forma q.a0 (uma raiz é 0), temos infinitas formas de escolher x = q.a0 de

[obm-l] Re: [obm-l] Re: [obm-l] equação

2002-11-14 Por tôpico Domingos Jr.
2) se a0 != 1 tome x = q.a0 com q inteiro, q != 0 fica simples verificar que a0|f(x) e, como f(x)= a0 = x é raiz de um polinômio de grau n (a1x +a2x^2 + anx^n) e esse polinômio pode ter no máximo n-1 raízes da forma q.a0 (uma raiz é 0), temos infinitas formas de escolher x = q.a0 de

[obm-l] Re: [obm-l] Re: [obm-l] Re: [obm-l] equação

2002-11-14 Por tôpico Domingos Jr.
Desculpem, o programa de envio de email estava indicado TIMEOUT do servidor SMTP e eu não sabia que a mensagem tinha sido mandada 3 vezes! = Instruções para entrar na lista, sair da lista e usar a lista em

[obm-l] Re: [obm-l] Teoria dos Números

2002-11-18 Por tôpico Domingos Jr.
1) ...1 ~ 1 mod 10 sabemos também que se a² ~ 1 mod 10 a ~ 1 ou a ~ 9 mod 10 caso a = 10x + 1 (10x + 1)² = 100x² + 20x + 1 = ..1 10x² + 2x = ...1 (com 299 dígitos) 2|10x² + 2x mas 2 não divide ...1 caso a= 10x + 9 (10x + 9)² = 100x² + 180x + 81 = ..1 isso vai dar 100x²

[obm-l] Re: [obm-l] ---- Questão IME

2002-11-19 Por tôpico Domingos Jr.
Ficaria muito agradecido se alguém me ajudasse na qustão do IME abaixo. -- Considere uma matriz A, nxn, de coeficientes reais, e k um número real diferente de 1. Sabendo que A^3=kA, prove que a matriz A+I é invertível, onde I é a matriz identidade nxn. (A + I)(A² - A + xI) = A³ - A² + xA

Re: [obm-l] PAs de ordens1

2002-11-22 Por tôpico Domingos Jr.
Estou num momento de diarréia mental. Qual é e como deduzir a fórmula de somatório de x^2, para x=1,2,..,n? pegue um polinômio p(n) tal que p(n) = somatório{ i = 1 até n } [ i² ] parece razoável esperar que esse polinômio tenha no máximo grau 3, já que p(n) n³ para todo n suponha então p(n)

Re: [obm-l] Matriz Inversa

2002-11-23 Por tôpico Domingos Jr.
Sejam A e X matrizes quadradas de ordem n e I a matriz identidade de mesma ordem. Para a equação: AX = I, posso afirmar que X é a inversa de A, ou é preciso definir que AX = XA = I Grato

Re: [obm-l] Re: PAs de ordens1

2002-11-23 Por tôpico Domingos Jr.
Antes de mais nada, obrigado pelas respostas do N para os raios e de todos que responderam às questões do somatório de x^2 e da PA de k-ésima ordem. Gostaria de comentar a resposta do Domingos Jr. em particular: Ou, mais genericamente, como se calcula a soma do n primeiros termos de uma

Re: [obm-l] Matriz Inversa

2002-11-23 Por tôpico Domingos Jr.
Laurito e demais colegas da lista, estruturando melhor minha pergunta fica assim: Hipótese: A e X são matrizes quadradas de orden n I denota a matriz identidade de mesma ordem. AX = I Tese:X é

Re: [obm-l] Matriz Inversa

2002-11-24 Por tôpico Domingos Jr.
Carissimos, voces estao supondo muito mais coisas do que o Daniel: o Daniel supunha apenas A quadrada e com inversa a direita. Laurito estah supondo que A tem inversa a direita e tem inversa a esquerda. Domingos, que A eh invertivel. Morgado Prof., se o enunciado nos diz que existe X tal que

Re: [obm-l] Matriz Inversa

2002-11-25 Por tôpico Domingos Jr.
Domingos, Acho que a colocacao do prof. Morgado foi muito bem feita. Seguindo a sua observacao, IMPLICITAMENTE estariamos afirmando que A possui inversa somente a direita e o problema nao afirmou que a matriz e quadrada. estavamos sim, assumindo que a matriz era quadrada: quote:

[obm-l] Re: [obm-l] Notícia

2002-12-01 Por tôpico Domingos Jr.
Legal :-) A questão (2) tem um jeito bem simples de ser resolvida: i) começa-se com um número ímpar de bolas brancas ii) as bolas brancas são sempre jogadas no lixo aos pares de (i) e (ii) temos que, a cada turno, o número de bolas brancas é ímpar. em especial, se no último turno só há uma

Re: [obm-l] ajuda

2002-12-04 Por tôpico Domingos Jr.
1, 16, 31, ..., 991... 6, 21, ...996, 11, 26, 986 ... 1 [pára antes de riscar o 1] - Original Message - From: [EMAIL PROTECTED] To: [EMAIL PROTECTED] Sent: Tuesday, December 03, 2002 10:30 PM Subject: [obm-l] ajuda Os inteiros de 1 a 1000 são escritos

[obm-l] Normas de matrizes

2002-12-09 Por tôpico Domingos Jr.
seja A = | A1 | | A2 | uma matriz m x n com A1 n x n não singular e A2 uma matriz (m-n) x n arbitrária ||.|| é a norma 2 sendo usada A+ é a pseudo-inversa de A, definida como A+ = (A'.A)^-1.A' prove que ||A|| = ||(A1)^-1|| / A norma aqui

Re: [obm-l] Normas de matrizes

2002-12-09 Por tôpico Domingos Jr.
é pra provar que: ||A+|| = ||(A1)^-1|| desculpem o erro! - Original Message - From: Domingos Jr. To: [EMAIL PROTECTED] Sent: Monday, December 09, 2002 4:16 PM Subject: [obm-l] Normas de matrizes seja A = | A1 | | A2 | uma matriz m x n com A1 n x n não

[obm-l] Re: [obm-l] RAIZ CÚBICA DE 7

2002-12-17 Por tôpico Domingos Jr.
a raiz cúbica de 7 é raiz de p(x) = x³ - 7 e p pertence a Q[X] p é irred. pelo critério de Eisenstein, para o primo 7 se p é irred., de grau 3 temos que raiz cúbica de 7 não pode pertencer a Q. Em livros sobre conjuntos numéricos, eles quase sempre apresentam uma prova por absurdo da

[obm-l] Re: [obm-l] Re: [obm-l] RAIZ CÚBICA DE 7

2002-12-17 Por tôpico Domingos Jr.
só no caso do polinômio ter grau = 3 pois, para graus maiores o fato de não haver raiz no corpo não quer dizer que o polinômio seja irred. nesse corpo. a sua pergunta depende muito de quem vai ler a resposta, se é para uma prova, pergunte parao seuprof. se é necessário dar mais detalhes na

Re: [obm-l] Probabilidade

2002-12-18 Por tôpico Domingos Jr.
Title: Help suponha que vc coloque x bolas brancas e y bolas pretas numa sacola no outra sacola devem haver 10-x b.b. e 10-y b.p. a probabilidade de pegar cada sacola é 1/2, logo a probabilidade de pegar uma bola branca é: (1/2).x/(x+y) + 1/2.(10-x)/[10-x+10-y] se as variáveis fossem

Re: [obm-l] Ajuda Sobre um site (estranho!!)

2002-12-20 Por tôpico Domingos Jr.
1) Pra gerar arquivos PS é muito simples, tanto no linux quanto no windows, é só mandar "imprimir para arquivo" através de uma impressora postscript. 2) Hosts para sites que não cobrem nada na Internet tem de monte, inclusive com e-mails... 3) Tem um site que tem um formato muito bom para

Re: [obm-l] Re:

2002-12-26 Por tôpico Domingos Jr.
Seja P(n) o numero de subconjuntos de 1,2,...,n com média inteira. Prove que P(n)-n é sempre par. um esboço, gostaria de receber comentários: seja T(i) o número de subconjuntos de {1, 2, ... i}, contendo i e com média inteira. seja A(i) a seguinte proposição: A(i) : P(i) - i é par e T(i+1) é

[obm-l] Re: [obm-l] Subconjuntos de {1,2,..,n} com Média Inteira

2003-01-02 Por tôpico Domingos Jr.
Olá, estive viajando e por tanto só estou lendo suas mensagens em 2003! (...) até aqui parece tudo bem... Seja X um elemento de @n com k elementos ( 1 = k = n ). No que se segue, vamos escrever X da seguinte forma: X = { A(1) , A(2) , ... , A(k) } e supor sempre que A(1) A(2) ... A(k).

Re: [obm-l] algoritmos

2003-01-04 Por tôpico Domingos Jr.
Acho que ficou extremamente vaga a sua pergunta Que tal fazer uma pergunta específica de árvore binária? um site interessante para pesquisar estruturas de dados e algoritmos: www.nist.gov/dads espero ter ajudado. - Original Message - From: [EMAIL PROTECTED] To:

Re: [obm-l] re:[(n^m) - n] multiplo de m ?

2003-01-05 Por tôpico Domingos Jr.
se m é primo mdc(n, m) = 1 ou m se mdc(n, m) = m então temos n = m.q para algum q inteiro, logo n^m - n = (mq)^n - m.q = m^n.q^n - m.q = m.(m^(n-1).q^n- q) logo m divide n^m - n se mdc(n, m) = 1 n !~ 0 (mod m) [ !~ quer dizer não congruente ] considere o anel dos inteiros módulo m, como m

Re: [obm-l] Conjuntos finitos

2003-01-15 Por tôpico Domingos Jr.
Seja F(X;Y) o conjunto das funcoes com dominio em X e imagem em Y. Se cardX=m e cardY=n, prove que cardF(X;Y)=n^m. seja f: X -- Y sejam x1, x2, ..., xm os elementos de X e y1, y2, ..., yn os elementos de Y f(x1) tem n possíveis valores f(x2) tem n possíveis valores ... f(xm) tem n possíveis

[obm-l] Re: [obm-l] Sequência 1, 3, 2, 6, 8, 4, 11, 5, 14

2003-01-16 Por tôpico Domingos Jr.
Acho que consegui resolver este problema... Algum voluntário pra verificar se a demonstração está correta? (espero que as imagens saiam legíveis) Seja a sequência X: N -- N (N = conjunto dos inteiros positivos), definida por: X(1) = 1, e, para n 1, X(n) = menor inteiro positivo tal que: (i)

Re: [obm-l] probabilidade

2003-01-19 Por tôpico Domingos Jr.
Por favor me ajudem nessa problema. Numa urna são depositadas n etiquetas numeradas de 1 a n .Tres etiquetas são sorteadas ( sem reposição).Qual a probabilidade de que os numeros sorteados sejam consecutivos? A primeira etiqueta sorteada pode ser 1...n-2. A segunda só pode ser uma das n - 1

Re: [obm-l] probabilidade

2003-01-19 Por tôpico Domingos Jr.
seja n = 3, a probabilidade então seria 6/(3.2) = 1??? acho que a probabilidade é 1/[n(n+1)] Ha C(n, 3) = n(n-1)(n-2)/6 modos de retirar 3 etiquetas e n-2 modos de retirar 3 etiquetas com numeros consecutivos [123, 234,..., (n-2)(n-1)n]. A resposta eh o quociente, 6/[n(n-1)] Morgado amurpe

Re: [obm-l] Probabilidade (Urna)

2003-01-19 Por tôpico Domingos Jr.
ganhar de primeira 1/3 + ganhar na segunda rodada... (2/3)².(1/3) + ganhar na terceira rodada... (2/3)^4.(1/3) + ... é uma somatória infinita... P = 1/3 + (4/9).(1/3) + (4/9)² + ... P*(4/9) = (4/9).(1/3) + (4/9)² + ... 5/9P = 1/3 P = 9/5 * 1/3 = 3/5 Boa noite lista, Gostaria da explicação da

[obm-l] funções elípticas e modulares

2003-01-19 Por tôpico Domingos Jr.
Gostaria de saber bons livros (em inglês ou português), conteúdo na Internet, qualquer coisa em nível introdutório. Se houver algum pré-requisito para entender esses conceitos eu também gostaria de sabê-los antes de começar. Obrigado!

Re: [obm-l] probabilidade

2003-01-19 Por tôpico Domingos Jr.
Para n=3 , a probabilidade é realmente igual a 1 , já que as etiquetas serão numeradas de 1 a 3 ( 123). []´s Carlos Victor Acho que entendi agora a diferença... o que eu calculei foi a retirada de uma etiqueta por vez e não das 3 etiquetas ao mesmo tempo... uma sutileza, mas eu

Re: [obm-l] probabilidade

2003-01-20 Por tôpico Domingos Jr.
e terceiro o 4); mas também poderia ter tirado 342 ,432,324,243,423 ; e da forma que você pensou essas sequências estariam presentes no seu espaço amostral , ok ? []´s Carlos Victor At 23:41 19/1/2003 -0300, Domingos Jr. wrote: Para n=3 , a probabilidade é realmente igual

[obm-l] Re: [obm-l] Sequência 1, 3, 2, 6, 8, 4, 11, 5, 14

2003-01-21 Por tôpico Domingos Jr.
Estou colocando a resolução em anexo PDF. [ ]'s seq.pdf Description: Adobe PDF document

[obm-l] Re: [obm-l] Re: [obm-l] Re: [obm-l] Sequência 1, 3, 2, 6, 8, 4, 11, 5, 14

2003-01-21 Por tôpico Domingos Jr.
Caro Domingos Jr. Dei uma primeira lida na sua demonstração e acho que a idéia funciona. Porém, tem uma passagem que não ficou clara: X(n) = m(n-1) + k.n para algum k inteiro Essa linha também nos diz que M(i) = {m(1), m(2), ... m(i)}está contido em {X(1), X(2),..., X(i+1)} pois o valor

[obm-l] Re: [obm-l] Sequências

2003-01-27 Por tôpico Domingos Jr.
seja x³ = x.x.x a² - b² = (a+b).(a-b) tome a + b = x² == a = x²- b a - b = x x² -2b -x = 0 x(x-1) = 2b b = x(x-1)/2 a + x(x-1)/2 = x² a = x(x+1)/2 a² - b² = (a+b).(a-b) = (x²).(x) = x³ - Original Message - From: Wagner To: [EMAIL PROTECTED] Sent: Monday, January 27,

[obm-l] Re: [obm-l] prova de uma afirmação

2003-02-01 Por tôpico Domingos Jr.
acho que sem a hipótese de f diferenciável realmente isso não é verdadeiro... dê uma olhada nessas funções que, apesar de serem contínuas, devem conter um intervalo fechado em que o valor de um extremo é maior que o outro e no entanto elas não possuem nenhum intervalo estritamente

[obm-l] Re: [obm-l] prova de uma afirmação

2003-02-03 Por tôpico Domingos Jr.
não seria bem isso, imagine que vc tem uma função que é toda cheia de subidas e descidas, mas a cada vez que sedá um "zoom" na função, algo que parecia uma reta crescente é na verdade um conjunto de subidas e descidas, e assim vai, essa função que eu descrevi de forma totalmente subjetiva

[obm-l] Re: [obm-l] Mximos e Mnimos SEM DERIVADAS

2003-02-06 Por tôpico Domingos Jr.
Sendo x um nº positivo determine o menor valor de E= 5x + 16/x + 21 E 21 para todo x 0 fixando o E, temos x != 0 E.x = 5x² + 16 + 21x 5x² + (21 - E).x + 16 = 0 x+ = [(E - 21) + sqrt[(E-21)² - 320]]/10 x- = [(E - 21) - sqrt[(E-21)² - 320]]/10 como x é positivo está implicito que ele é real e

Re: [obm-l] Como se leva algo as ultimas consequencias

2003-02-07 Por tôpico Domingos Jr.
não vai ao menos nos dizer qual é a solução do Tengan? - Original Message - From: Johann Peter Gustav Lejeune Dirichlet To: [EMAIL PROTECTED] Sent: Friday, February 07, 2003 12:40 PM Subject: [obm-l] Como se leva algo as ultimas consequencias Esse e-mail

[obm-l] Re: [obm-l] Re: [obm-l] cálculo

2003-02-09 Por tôpico Domingos Jr.
dá pra complicar e resolver usando integrais duplas também :-p considere a base quadrada e tome f(x, y) uma função definida na região do plano xy correspondente que leva o ponto (x, y) da base ao ponto da superfície da pirâmide. Volume = IntDupla{ f(x, y) dxdy } na região do quadrado. supondo

[obm-l] Re: proposta para a lista

2003-02-11 Por tôpico Domingos Jr.
Aproveito a oportunidade para fazer uma outra proposta, já que há outra discussão em torno da lista --- se deve ser criada uma lista paralela ou não. Acredito que muitos daqui já tenham utilizado newsgroups, tornar essa lista de discussão um newsgroup seria muito vantajoso e acredito que não seja

[obm-l] quase-solução do problema da imo87

2003-02-11 Por tôpico Domingos Jr.
Acho que está quase lá, consegui limitar muito o intervalo onde f pode ser composto. Espero que não passe o limite dos 20k chars. imo.zip Description: application/compressed

Re: [obm-l] Tres belos problemas

2003-02-11 Por tôpico Domingos Jr.
1) Caracterize todas as PA's nas quais qualquer soma de um numero qualquer de termos consecutivos e ainda um termo desta PA. Seja a, a + r, a + 2r uma PA e a[i] = a + i.r S = somatório { de i = j até k } a[i] = (k-j+1).a + r * somatório { de i = j até k } i Se S pertence a { a[0], a[1],

[obm-l] Re: [obm-l] Re:_[obm-l]_x^2+x+p_é_primo

2003-02-14 Por tôpico Domingos Jr.
Veja a soluçao no final do e-mail mas se matem primeiro. "Domingos Jr." [EMAIL PROTECTED] wrote: pelo que eu vi na prova da IMO que eu baixei é raiz quadrada... pelo menos alguém leu o que eu fiz!!! - Original Message - From:

Re: [obm-l] Bibliotecas..

2003-02-19 Por tôpico Domingos Jr.
Acho que ele estava falando do IME militar e não da Usp... Aliás, o que vc faz lá? Eu faço ciência da computação... - Original Message - From: Johann Peter Gustav Lejeune Dirichlet To: [EMAIL PROTECTED] Sent: Wednesday, February 19, 2003 3:56 PM Subject: Re:

Re: [obm-l] Oi Pessoal

2003-02-20 Por tôpico Domingos Jr.
algumas idéias... (http://mathworld.wolfram.com/EulersTotientTheorem.html) phi(10^1000) é o número de inteiros de 1...10^1000 que são relativamente primos com 10^1000. temos que todos os múltiplos de 2 ou 5 são os únicos inteiros com divisor em comum com 10^1000, logo, o número de múltiplos de 2

Re: [obm-l] Treta nos quadrados

2003-02-24 Por tôpico Domingos Jr.
já que vc provou para o caso4 vou considerar a demonstração feita. estou considerando as variáveis todas positivas... se não, teríamos, por exemplo (1 + 1 + 0 + 0 + 0 + ... 0- 1 - 1)² = 0 4*(1*1 + 1*0 + 0*0 + ... + 0*(-1) + (-1)*(-1) + (-1)*1) = 4 suponha que a desigualdade valha para

Re: [obm-l] SIGNIFICADO.

2003-02-24 Por tôpico Domingos Jr.
se vc pegar um livro de álgebra como o do Fragleigh vc encontra um diagrama mostrando os anéis da álgebra, vou tentar explicar o diagrama: quando eu escrever anel entre aspas estará sendo no sentido de figura de um anel e não da estrutura algébrica. temos anéis comutativos de um lado e anéis com

[obm-l] Re: [obm-l] Algum progresso (seqüência de potências)

2003-02-25 Por tôpico Domingos Jr.
Para o caso mdc(a, 10) = 1, basta então provarmos que existe n tal que A(n+1) = A(n) (mod 10^1002) teremos provado que os 1000 últimos dígitos eventualmente são fixados... Acho que consegui fechar a prova para o caso mdc(a, 10) = 1. defina g(n) = A(n+1) - A(n), n = 1 manipulando g(n): g(n) =

Re: [obm-l] Desigualdade

2003-02-26 Por tôpico Domingos Jr.
mximo 1/4. a demonstrao da desigualdade eu provei por induo numa outra mensagem pra lista. - Original Message - From: Cludio (Prtica) To: [EMAIL PROTECTED] Sent: Monday, February 24, 2003 2:43 PM Subject: [obm-l] Desigualdade Caros JP, Domingos Jr. e Artur: S pra

[obm-l] Re: [obm-l] Algum_progresso_(seqüência_de_potências)

2003-02-26 Por tôpico Domingos Jr.
te garante isso inicialmente. "Domingos Jr." [EMAIL PROTECTED] wrote: Para o problema:seja A(1) = a e A(n + 1) = a^A(n) para n = 1, provar que para todo inteiroa 1 os últimos 1000 dígitos da série A(1), A(2), ... eventualmente semantém fixos.seja a tal que mdc(

[obm-l] Re: [obm-l] Algum_progresso_(seqüência_de_potências)

2003-02-26 Por tôpico Domingos Jr.
Sinceramente, acho que você não deveria colocar um problema desses na categoria dos triviais, primeiro porque o nível dos participantes não é homogêneo e é para muitos esse é um problema difícil. Confesso que perdi um bom tempo para sacar que era possível aplicar o tma. de Euler repetidamente ao

[obm-l] Re: [obm-l] Dúvida de vestibular

2003-02-26 Por tôpico Domingos Jr.
Oi Pessoal, Estava estudando análise combinatória por uma apostila de um curso pré-vestibular, e encontrei o seguinte problema, que achei interessante, mas minha solução foi muito longa, e não sei se está certa, porque tinha muitos casos. Se estivesse num vestibular, o que faria?

Re: [obm-l] Bem vindo OKAKAMO

2003-02-26 Por tôpico Domingos Jr.
1) acho que d pra resolver assim: prove que para polinmios quaisquer de 1 varivel o nmero desolues mltiplo de q suponha que para polinmioscom nmero devariveis 1 = k = n isso vale pegue um polinmio de k+1 variveis p(x1, x2, ..., xk, x[k+1]) os valores possveis para x[k+1] so { 0, 1, 2,

Re: [obm-l] Probleminhas legais

2003-03-05 Por tôpico Domingos Jr.
1. O produto de alguns primos é igual a 10 vezes a soma desses primos. Quais são esses primos( não necessariamente distintos)? sejam p1, p2 ... p[n] tais primos: (p1 + p2 + ... + p[n]).2.5 = p1.p2.p3...p[n] assuma sem perda de generalidade então que p1 = 2 e p2 = 5 já que esses dois primos

Re: [obm-l] Probleminhas legais

2003-03-05 Por tôpico Domingos Jr.
2. Considere f:Q - Q tal que f(x + f(y)) = f(x).f(y) para todo x,y pertencentes a Q. Prove que f é constante. seja f : Q - Q f(x + f(y)) = f(x).f(y) i) suponha que existe x0 tq. f(x0) = 0 f(a + f(x0)) = f(a + 0) = f(a) f(a + f(x0)) = f(a).f(x0) = f(a).0 = 0 logo f(a) = 0 para todo a, sendo

Re: [obm-l] (O (sqrt n))

2003-03-06 Por tôpico Domingos Jr.
Quando eu mando o Maple fazer uma série de Taylor para uma função, aparece esse O também. Creio eu que seja algo muito pequeno, um infinitesimal, não sei direito. Henrique. Não necessariamente, a notação O na verdade significa: f(x) = O(g(x)) = existe uma constante c que, para todo x x0,

[obm-l] Re: [obm-l] Re: [obm-l] Funções booleanas

2003-03-06 Por tôpico Domingos Jr.
Interessante a sua prova por indução. Pra mim, o seguinte argumento já seria convincente: exatamente como o Wendel disse, a prova em si não é o fato de que ao criar um conectivo ele pode ser expresso como uma combinação desses 3, mas sim provar que toda função pode ser expressa com eles (como o

Re: [obm-l] Primos numa PA

2003-03-10 Por tôpico Domingos Jr.
o máximo que eu cheguei é que dado qualquer a natural não nulo, deve existir um b tal que {an + b / n natural} contém infinitos primos... isso sai de maneira bem simples, tome o conjunto de todos primos e verifique sua congruência módulo a, obviamente não podemos ter todas as classes de

[obm-l] Re: [obm-l] 2ª Vingança Olimpica-Prova

2003-03-12 Por tôpico Domingos Jr.
2)(Alex Abreu)Defina a sequencia x(1) natural e x(n+1)=1+(x(1)x(2)x(3)...x(n)). Prove que existe um primo p que nao divide ninguem da sequencia acima.[4] para n 1 x(n) = 1 + x(1).x(2)...x(n-1) = x(1).x(2)...x(n-1) = x(n) - 1 x(n + 1) = 1 + x(1)...x(n-1).x(n) = 1 + x(n)[x(n) - 1] = x(n)² -

[obm-l] Re: [obm-l] 2ª Vingança Olimpica-Prova

2003-03-13 Por tôpico Domingos Jr.
2)(Alex Abreu)Defina a sequencia x(1) natural e x(n+1)=1+(x(1)x(2)x(3)...x(n)). Prove que existe um primo p que nao divide ninguem da sequencia acima.[4] Consegui resolver o problema com uma ajuda do Wendel, mas não tenho como postar o pdf com a resolução pois o arquivo ultrapassa os 20k e,

[obm-l] Re: [obm-l] Re: [obm-l] Re: [obm-l] 2ª Vingança Olimpica-Prova

2003-03-13 Por tôpico Domingos Jr.
Aumentar o limite faria com que um monte de vírus (que hoje são barrados) entrassem na lista. Estranho... vírus são arquivos pequenos, passariam pelo limite de 20k numa boa... acredito que vc esteja falando das tranqueiras como Love Letter e outros scripts mal intencionados. A pasta que você

[obm-l] Re: [obm-l] Re: [obm-l] Irã [1999]

2003-03-18 Por tôpico Domingos Jr.
suponha que 2^m e 2^n são números com os dígitos rearranjados, sem perda de generalidade, assuma 2^m 2^n, é evidente que n = m+1, m+2 ou m+3, pois para m+4 o número 2^n tem necessariamente pelo menos um dígito a mais que 2^m! se os dígitos são os mesmos, é fácil verificar que a congruência deles

Re: [obm-l] Provar primo impar

2003-03-22 Por tôpico Domingos Jr.
ei pessoal, como é que eu provo que qualquer número primo impar pode ser escrito ou da forma 4n + 1 ou 4n - 1 ?? só existem números inteiros da forma 4n, 4n + 1, 4n + 2, 4n + 3 note que 4n e 4n + 2 são pares e por tanto um primo ímpar não pode se escrever dessa forma... mas 4n + 3 = 4(n+1) -

Re: [obm-l] Problema

2003-03-25 Por tôpico Domingos Jr.
Também tenho quebrado a cabeça com ele... Uma primeira idéia foi considerar que existe uma divisão em 6 partições onde nenhum elemento é soma de outros dois pertencendo a mesma partição e, para cada partição, definir um conjunto de elementos que NÃO podem entrar na partição, pois se entrasse

Re: [obm-l] Problema

2003-03-29 Por tôpico Domingos Jr.
Nao sei se eh da IMO ou nao. Eu vi num artiga da Eureka sobre o principio das gavetas e a solucao do Morgado era essencialmente a mesma. Ah, o Morgado já respondeu, IMO 1978, tinha cara mesmo... Alias, eu to bem enrolado na questao dos doces de jaca e jilo da vinganca olimpica, apesar das

Re: [obm-l] Mais Problemas em Aberto

2003-03-31 Por tôpico Domingos Jr.
Title: Help p, o 7.2 eueo Wendelj provamos: http://www.linux.ime.usp.br/~domingos/problema.ps http://www.linux.ime.usp.br/~domingos/problema.pdf

Re: [obm-l] Mais Problemas em Aberto

2003-04-01 Por tôpico Domingos Jr.
Consegui estimar um limitante inferior para o nmero de grupos de crianas: Considere uma matriz com elementos A[i, j] = (i, j) pertence a (Zp) O problema proposto equivalente a calcular o nmero de combinaes de elementos de A cuja soma d (0, 0). Agora desenhando a matriz A e separando a ltima

Re: [obm-l] Mais Problemas em Aberto

2003-04-02 Por tôpico Domingos Jr.
D pra melhorar bastante esse limitante: A idia baseia-se no seguinte fato: todo inteiro entre 1...2^(n+1)-1 pode ser expresso como soma de elementos de uma combinao de {1, 2, 2, ..., 2^n}. Seja k um inteiro tal que 2^(k-1) p 2^k Da matriz A j definida, separe os elementos: S1 = {(1, 0); (2,

Re: [obm-l] O problema do andarilho

2003-04-02 Por tôpico Domingos Jr.
Alguem poderia me ajudar com esse? Uma trilha vai da base de uma montanha até o topo. Um andarilho começa a subir a trilha às 6 horas da manhã e chega ao topo às 6 horas da tarde do mesmo dia. Durante o percurso ele pode parar, voltar atrás, correr, fazer o que quiser desde que chegue ao

Re: [obm-l] provar igualdade

2003-06-06 Por tôpico Domingos Jr.
O que você provou é o sentido contrário do que estava sendo pedido... o que vc provou é que se a = b = c então a^2 + b^2 + c^2 = ac + bc + ab, o que é, como você mesmo observou, trivial. Quando você está aprendendo conceitos elementares da matemática é muito comum confundir os 'sentidos' da

Re: [obm-l] RE problema real

2003-06-06 Por tôpico Domingos Jr.
Essa fórmula é um saco, mas pra um computador é só mais uma fórmula! http://www.math.nmsu.edu/~history/book/cardano.pdf na verdade a fórmula foi descoberta bem antes de Cardano, mas... o nome pegou! Aos amigos Claudio, Domingos e Joa Gilberto, agradeco a colaboracao. Vou testar as sugestoes

Re: [obm-l] Primos numa PA

2003-06-06 Por tôpico Domingos Jr.
HelpEstamos analisando a congruncia de primos mod m. Suponha que o conjunto de primos que so congruentes a b mod m finito e seja P = {p1, ..., p[k]} tal conjunto, e alm disso P != . note que mdc(m, b) = 1 [aqui usamos a hiptese da existncia de am + b = primo] tome Q como um conjunto gigante de

Re: [obm-l] derivada de ordem 1000

2003-06-06 Por tôpico Domingos Jr.
Para isso preciso do milésimo coeficiente. Ora, se c[0] = 1 c[1] = x^2 c[2] = (x^4)/2! c[3] = (x^6)/3! Por intuicao creio que c[n] = (x^2n)/n! Então c[1000] = (x^2000)/1000! (II) Substituindo I em II vem P(1000)(0) = (1000!)(0^2000)/1000! P(1000)(0) = 0.

Re: [obm-l] Primos numa PA

2003-06-07 Por tôpico Domingos Jr.
tome agora o nmero n = produtrio {t, t pertencendo a Q - {primos divisores de m}} + m + b no funciona, no d pra garantir que primo e nem era bem isso que eu queria dizer... qdo eu estiver com menos sono eu penso melhor. [ ]'s

[obm-l] Re: [obm-l] Re: [obm-l] Olimpíadas ao redor do mundo....

2003-05-30 Por tôpico Domingos Jr.
Cláudio, eu tive a mesma idéia que você, mas havia expressado de outra forma... seja p um primo tal que a ~ 1 (mod p) a menos de a = 2, esse primo existe, basta pegar um primo que divida a - 1 (como vc bem notou, esse primo existe sempre para a 2). agora note que 1 + a + ... + a^(p-1) ~

  1   2   3   4   5   >